- Tue Jun 14, 2016 12:54 pm
#26549
Complete Question Explanation
(The complete setup for this game can be found here: lsat/viewtopic.php?t=11086)
The correct answer choice is (B)
Answer choice (A) is incorrect because M cannot be ranked last.
Answer choice (C) is incorrect because if J is ranked third, then from the fourth rule H must be ranked first, a violation of the second rule.
Answer choice (D) is incorrect because when K is ranked second then L must be ranked fifth, and when H is ranked third, then J must be ranked fifth, and according to the first rule there are no ties.
Answer choice (E) is incorrect because there is no available ranking for O when M is seventh and L is eighth.
Accordingly, answer choice (B) is proven correct by elimination.
(The complete setup for this game can be found here: lsat/viewtopic.php?t=11086)
The correct answer choice is (B)
Answer choice (A) is incorrect because M cannot be ranked last.
Answer choice (C) is incorrect because if J is ranked third, then from the fourth rule H must be ranked first, a violation of the second rule.
Answer choice (D) is incorrect because when K is ranked second then L must be ranked fifth, and when H is ranked third, then J must be ranked fifth, and according to the first rule there are no ties.
Answer choice (E) is incorrect because there is no available ranking for O when M is seventh and L is eighth.
Accordingly, answer choice (B) is proven correct by elimination.